if f(x)=1/9x-2, what is f^-1(x)?​

Answers

Answer 1

Answer:

[tex]f^{-1}(x)[/tex] will be equal to 9y+18

Step-by-step explanation:

[tex]f^{-1}(x)[/tex] will be equal to 9y+18


Related Questions

A bag contains 9 tiles, each with a different number from 1 to 9. You choose a tile without looking, put it aside, choose a second tile without looking, put it aside, then choose a third tile without looking. In how many different ways can you choose three tiles

Answers

Using the combination formula, it is found that there are 84 ways to choose the three tiles.

The order in which the tiles are chosen is not important, hence the combination formula is used to solve this question.

What is the combination formula?

[tex]C_{n,x}[/tex] is the number of different combinations of x objects from a set of n elements, given by:

[tex]C_{n,x} = \frac{n!}{x!(n-x)!}[/tex]

In this problem, three tiles are chosen from a set of nine, hence:

[tex]C_{9,3} = \frac{9!}{3!6!} = 84[/tex]

There are 84 ways to choose the three tiles.

More can be learned about the combination formula at https://brainly.com/question/25821700

hey who knows this? i need help

Answers

Answer:

I don't see any thing there

Step-by-step explanation:

u need to include the picture

J^3 = 0.125 (I don’t understand help me please)

Answers

[tex]~\hspace{7em}\textit{negative exponents} \\\\ a^{-n} \implies \cfrac{1}{a^n} ~\hspace{4.5em} a^n\implies \cfrac{1}{a^{-n}} \\\\[-0.35em] \rule{34em}{0.25pt}\\\\ J^3=0.125\implies J^3=\cfrac{0125}{1000}\implies J^3=\cfrac{1}{8}\implies J^3=\cfrac{1}{2^3} \\\\\\ J^3=2^{-3}\implies J^3=(2^{-1})^3\implies J=2^{-1}\implies J=\cfrac{1}{2}[/tex]

Ann and Makiko like to swim laps at the community center pool.

They are swimming together today (day 1), but they are on different swim schedules. Ann swims every three days and Makiko swims every five days.

Question: How many times will they both swim at the pool on the same day
during the next ten weeks (including the first day)?

please explain process and show thinking in steps.

Answers

Answer:   5

=========================================================

Explanation:

The LCM of 3 and 5 is 3*5 = 15.

Every 15 days, the two people will swim together on the same day.

Day 1 = first day swimming togetherDay 1+15 = Day 16 = second day swimming togetherDay 16+15 = Day 31 = third day swimming together

And so on.

Keep this process going of adding on 15 to find this sequence

1, 16, 31, 46, 61, 76, ...

We stop the sequence once we either reach 70 or go over it because there are 7*10 = 70 days in ten weeks. This assumes we use a 7 day week rather than a 5 day week.

As shown in bold, there are 5 items smaller than 70. So the two people swam together a total of 5 times

-----------------

Alternatively, you can solve 15n+1 = 70 for n to get n = 4.6 which rounds down to n = 4 to represent the set {0,1,2,3,4} which has 5 items in it to represent the five times total they swim together.

n = 0 corresponds to day 15n+1 = 15(0)+1 = 1n = 1 corresponds to day 15n+1 = 15(1)+1 = 16

and so on until reaching n = 4 corresponding to day 15(4)+1 = 61.

Mr. Garcia has 15 boys and 10 girls in his math class. He selects wo students at random to demonstrate how they solved the day's challenge assignment What is the probability that both students are girls?​

Answers

Answer: 3/20

Step-by-step explanation:

3 3/4 ÷ (1 2/3 + 2 1/2)​

Answers

Answer:

9/10.

Step-by-step explanation:

3 3/4 ÷ (1 2/3 + 2 1/2)​

= 3 3/4 / ( 5/3 + 5/2)

= 3 3/4 / ( 10/6 + 15/6)

= 3 3/4 / 25/6

= 15/ 4 / 25/6

= 15/4 * 6/25

= 90/100

= 9/10.

How do you write 1.27 repeating as a fraction

btw the repeating part is 7

Answers

Answer:

1 27/100

Step-by-step explanation:

A plane flies 100km north and then flies 400km south, what is its new position?

Answers

Answer:

300km south

Step-by-step explanation:

Go up once from original position then down four times

Which expression is equivalent to 3
5v ?

Answers

Answer:

x ^5/3 y ^1/3 (the second option listed)

Step-by-step explanation:

So without even solving the expression, you can find the correct option. If you're finding the cube root (or really any square root), you know it will be less than your original number (unless you are working with imaginary numbers, but you aren't).

So, if you are finding the cube root of an expression, you're finding the cube root of each number separately.

You wouldn't be cubing y, you would be finding the cube root. You wouldn't be leaving y the same, you are finding the cube root.

This means that without even solving (which I will explain),

the correct option is the second one on the screen.

Here's how to actually solve the equation:

So, if we are finding the cube root of a variable with an exponent, we can divide that exponent by 3 (because it is a cube root).

When we have a square root, you know that we are breaking down each number/variable into an even exponent to solve the expression. When taking the square root of an even exponent, such as 4, you would divide by 2.

This same strategy applies to cube roots. You are dividing the exponent of both variables by 3.

[remember: the exponent of a variable without an exponent is essentially an invisible exponent of 1.]

What is the value of 6(2b-4) when b=5?

Answers

Answer:

Step-by-step explanation:

What is the value of 6(2b-4) when b =5 2.  The value of is 36. Step-by-step explanation: Given expression: To find the value of at b= 5, we need to substitute the b=5 in the expression, we get.

can i have brainly please :) thanks bro

Answer: 36

Step-by-step explanation:

Describe the relationship between pi and circles.

Answers

Answer:

The circumference (C) of a circle is found by the equation C = 2*[tex]\pi[/tex]*radius

There is a second relationship between [tex]\pi[/tex] and circles.  That of the area (A) of the circle which is found by the formula A = [tex]\pi*radius^{2}[/tex]

Step-by-step explanation:

(05.02)
A line passes through (2, 8) and (4, 12). Which equation best represents the line?
oy-x+6
=
2
y = 2x + 7
y = 2x + 4
oy-*x+10
=

Answers

Answer:

⊕ y=2x+4

Step-by-step explanation:

1) all the answers are provided in slope-interception form, then

2) the slope of the required line is: (12-8)/(4-2)=2. The required line is like to y=2x+??;

3) if the points (2;8) and (4;12) belong to the line required, then the point (0;4) also belongs to this one. It means, the interception is 4;

4) finally, the line required is y=2x+4.

PS. note, the suggested solution is not the only way.

Save my soul!!!!
if you can pIz show me what you did to get answers.

Answers

Answer:

9

41/4

23/2

51/4

14

Step-by-step explanation:

The first term is the first term that is given. Just copy it.

Each next term is the previous term plus the common difference.

First term: a_1 = 9

Second term: a_2 = 9 + 5/4 = 36/4 + 5/4 = 41/4

Third term: a_3 = 41/4 + 5/4 = 46/4 = 23/2

Fourth term: a_4 = 46/4 + 5/4 = 51/4

Fifth term: a_5 = 51/4 + 5/4 = 56/4 = 14

find the value of x, round to the nearest 10th

I will give brainliest

Answers

ACB + BAC =90
ACB + 37 = 90
ACB = 53
Tan (53) = 10/x
X= 7.536
(Hope this helped have a great day)

100 points and brainliest if u answer this right


Consider this dilation

(a) Is the image of the dilation a reduction or an enlargement of the original figure? Explain.

(b) What is the scale factor? Explain.

Answers

Answer:

a) Enklargement, b) k = 2

Step-by-step explanation:

a)

The original figure is ABC and the image is A'B'C'.

We can see the image is larger than the original figure.

It indicates that the dilation is an enlargement.

b)

Find the ratio of corresponding sides to find the scale factor.

First get the measure of corresponding line segments.

AC = 2 - (-2) = 4A'C' = 5 - (-3) = 8

The scale factor is:

k = A'C'/AC = 8/4 = 2

#1

Enlargement

#2

Take two sides of both

BC=|-3-1|=4unitsB'C'=|-3-5|=8units

Scale factor:-

k=8/4=2

Question 1-3
The graph below shows a function that represents the weekly earnings, in dollars, for a company that makes x sets of headphones per week.
7,0001
6,000
5,000
4,000
Earnings (5)
3,000
2,000
1,000
0 1,000 2,000 3,000 4,000 5,000 6,000
Number of Headphones Made per Week
Based on this model, which statements are true? Choose ALL that are correct.
When producing 5,000 sets of headphones per week, the weekly earnings is $0.
If production is increased from 3,000 to 4,000 sets of headphones per week, then the weekly earnings will also increa
Producing 2,500 sets of headphones per week will give the maximum amount of weekly earnings.
The maximum possible earnings is about $6,250 per week.
If production is increased from 1,500 to 2,000 sets of headphones per week, then the weekly earnings will also increa

Answers

The production graph of the company is a quadratic graph

The true statements about the production graph are:

When producing 5,000 sets of headphones per week, the weekly earnings is $0.Producing 2,500 sets of headphones per week will give the maximum amount of weekly earnings.The maximum possible earnings is about $6,250 per week.If production is increased from 1,500 to 2,000 sets of headphones per week, then the weekly earnings will also increase

How to interpret the model?

To interpret the model, we simply evaluate the options one after the other

Producing 5000 sets weekly

From the graph, we have:

(x,y) = (5000,0)

This means that a weekly production of 5,000 sets of headphones give an earning of $0.

Hence, this option is true

Increased production from 3000 to 4000

From the graph, the function value decreases when x increases from 3000 to 4000.

This means that this option is false

Producing 2500 sets

The production of 2500 sets give a maximum earning of $6250

This means that options (c) and (d) are true

Increased production from 1500 to 2000

From the graph, the function value increases when x increases from 1500 to 2000.

This means that this option is true

Hence, the true statements are: (a), (c), (d) and (e)

Read more about quadratic graphs at:

https://brainly.com/question/7988424

In ΔJKL, the measure of ∠L=90°, KJ = 41, JL = 40, and LK = 9. What ratio represents the cosine of ∠J?

Answers

1 because if you were to get h rn e measure of the problem right here it would be a cosine of 1

Consider the incomplete paragraph proof.

Given: P is a point on the perpendicular bisector, l, of MN.
Prove: PM = PN

Line l is a perpendicular bisector of line segment M N. It intersects line segment M N at point Q. Line l also contains point P.

Because of the unique line postulate, we can draw unique line segment PM. Using the definition of reflection, PM can be reflected over line l. By the definition of reflection, point P is the image of itself and point N is the image of ________. Because reflections preserve length, PM = PN.

point M
point Q
segment PM
segment QM

Answers

Answer: point M

Step-by-step explanation:

Answer: point M

i took the test

Y^2+8y+2 y=-5 evaluate the expression

Answers

Answer:

the correct answer is shown below

Step-by-step explanation:

hope this helps

have an awesome day -TJ

pls mark brainliest if this helps :)

Write an exponential function for the graph

A. Y=0.5(2)x
B. Y=2(0.5)x
C. Y=(2•0.5)x
D. Y=2(5)x

Answers

Answer: c

Step-by-step explanation: Simple addition with decimals.

Evaluate expression if a = 2.

a² - 5a - 6

Answers

Answer:

-12.

Step-by-step explanation:

a^2 - 5a - 6

When a = 2 we have

2^2 - 5(2) - 6

= 4 - 10 - 6

= -12.

Answer:

Step-by-step explanation:

a²- 5a - 6

Substitute a for 2

(2)² - 5(2) - 6

Solve.

4 - 10 - 6 = -12

If a = 2, a²- 5a - 6 equals -12

Pls pls pla help it lol

Answers

66x is the correct answer

Sectionalism:Question 10
What was the main reason that most northerners were opposed to the
new Fugitive Slave Act created by the Compromise of 1850?

Answers

Answer:

They felt it gave the commissioners too much power and was unfair to slaves by giving them a trail without injury

i need help with the second part of the question please helppp!!! will give brainliset

Answers

Answer:

$79.99

Correct method:

[tex]\rightarrow \sf 5(r-14.5)=327.45[/tex]

[tex]\rightarrow \sf 5r-72.5=327.45[/tex]

[tex]\rightarrow \sf 5r=327.45+72.5[/tex]

[tex]\rightarrow \sf 5r=399.95[/tex]

[tex]\rightarrow \sf r=\dfrac{399.95}{5}[/tex]

[tex]\rightarrow \sf r=79.99[/tex]

The regular price is $79.99

Answer:

$79.99

Step-by-step explanation:

The error is in step 3.  He needed to ADD 72.50 to both sides of the equation.  Instead, he added 72.50 to the left side and subtracted 72.50 from the right side.

Correction calculation

5(r - 14.50) = 327.45

5r - 72.50 = 327.45

             5r = 399.95

                r = 79.99

The regular price is $79.99        

The question is below:

Answers

QUESTION -> {WHICH EQUATION DESCRIBES A LINEAR FUNCTION }.

?

ANSWER :

B .

y =(1 6 ) ×

or.

C .

y = (2) ×

HOPE IT HELPS .

THANK ME LATER

THANKS.

Identify the equation in slope-intercept form for the line containing the points
(3,5) and (2,-5)
Choices:
y = 3x − 15
y = 5x − 25
y = 10x − 25
y = 5x + 10
Pls help

Answers

Answer:

Third Option y=10x-25

Step-by-step explanation:

Identify the equation in slope-intercept form for the line containing the points

(3,5) and (2,-5)

Choices:

y = 3x − 15

y = 5x − 25

y = 10x − 25

y = 5x + 10

Pls help

Sooo let’s solve this….

Notice the x-axis

Slope Intercept form changes at a constant rate soooo… from (3,5) to (2,-5)

The axis goes left by 1

On A graph the more to the left the smaller number get.

The Y-axis goes down by 10 from the bigger point

But…..

It’s really not going left 1 and 10 down you always start with the smaller point so from

(2,-5) to (3,5) It goes Right 1 and 10 up

Knowing this

Slope Intercept form

Y=mx+b

Where m=slope

b= y-intercept where the line crosses x=0

Our Slope Is 10

Because from -5 to 5 you have to add 10 to get the other value

y=10x+b

But lets just figure out the y-intercept to be sure.

We want the X-axis to be 0 so we will be going left until we get to 0

So We get this point (2,-5)

Knowing that we go up 10 and right 1 to get to the higher value we will do the opposite.

We will go 1 to the left and 10 down

(2,-5)

(1,-15)

(0,-25) As you can see x=0 so the y-intercept will be -25

y=mx+b

m=slope

b= y-intercept

y=10x-25 The Third Option is the Answer….

SonicIsCoool, at your service if you need any help :)

PLS HELP ME 50 POINTS AND I WILL MARK THE BRAINLIEST

Answers

Answer:

1. 392 in², 2. 94.99 cm², 3. 145.2 ft², 4. 161.5 m², 5. 17 ft and 81 cm

Step-by-step explanation:

1. A = 16 x 19

A = 304

A = 1/2(11x16)
A = 1/2(176)
A = 88
304 + 88 = 392 in²

2. A = πr²
A = 3.14*5.5²
A = 3.14*30.25
A = 94.985
A = 94.99 cm²

3. A = 1/2(bh)
A = 1/2(11x26.4)
A = 1/2(290.4)
A = 145.2 ft²

4. A = bh
A = 9.5*17
A = 161.5 m²

5. part 1
A = (1/2) · (p + q) · h

112 = (1/2)(11+q)(8)
112 = 4(11+q)
112 = 44 + 4q
68 = 4q
17 = q

17 ft

5. part 2
A = lw
364.5 = 27w
13.5 = w
P = 2l + 2w
P = 2(27) + 2(13.5)
P = 54 + 27
P = 81
81 cm


Hope this helps :)


Find the derivative of tan(x).

Answers

[tex]\qquad\qquad\huge\underline{{\sf Answer}}♨[/tex]

Derivative of tan(x) is sec²x

[tex]\qquad \sf  \dashrightarrow \: \therefore \dfrac{d}{dx} ( \tan(x)) = { \sec}^{2} (x)[/tex]

You can check the first principle method of derivation in attachment

[tex]\rightarrow \sf \dfrac{d}{dx} (tan(x))[/tex]

[tex]\rightarrow \sf \dfrac{d}{dx} ( \ \dfrac{sin(x)}{cos(x)} \ )[/tex]

use the quotient rule

[tex]\rightarrow \sf \dfrac{cos(x) * \dfrac{d}{dx} (sin(x)-sin(x)*\dfrac{d}{dx}(cos(x) }{cos(x)^2}[/tex]

[tex]\rightarrow \sf \dfrac{cos(x) * cos(x)-sin(x)*(-sin(x) )}{cos(x)^2}[/tex]

[tex]\rightarrow \sf \dfrac{cos(x)^2+sin(x)^2}{cos(x)^2}[/tex]

[tex]\rightarrow \sf \dfrac{1}{cos(x)^2}[/tex]

[tex]\rightarrow \sf sec(x)^2[/tex]

used formula's :

cos²(x) + sin²(x) = 1[tex]\sf \dfrac{1}{cos^2(x) }= sec^2(x)[/tex][tex]\sf \frac{d}{dx}[/tex] cos(x) = -sin(x)[tex]\sf \frac{d}{dx}[/tex] sin(x)  = cos(x)tan(x) = sin(x)/cos(x)

Help
Arc JK = ______°Arc

M<1 = ______°

Answers

Answer:

Step-by-step explanation:

The decimal value of 5/8

Answers

0.625

Explanation:
We know that 0.125 is equal to 1/8, so you can then multiply 0.125 by 5.

Dividing 5/8, It's Equal To 0.625

Step-by-step explanation:

5/8 = O.625

Other Questions
Find the volume. Use 3.14 for where necessary. Round the answer to two decimal places. laura can use a toy car to demonstrate the force of gravity by doing which of these PLS ANSWER IT I just need the answer People use arpilleras as bedspreads. true or false A square aluminum plate 5 mm thick and 150 mm on a side is heated while vertically suspended in quiescent air at 75c. determine the average heat transfer coefficient for the plate when its temperature is 15c by two methods: using results from the similarity solution to the boundary layer equations, and using results from an empirical correlation. Serafina and the Black CloakTrue or False: One big clue that leads serafina to believe that mr. throne is the man in the black cloak is that he has too many talents and seems too good to be true. The female gametophyte is a _______ structure called an embryo sac, while the male gametophyte is a _______ structure called a pollen grain. When pollen attaches to the stigma of a flower, one of the sperm cells fertilizes an egg cell, producing a _______ zygote. Another sperm cell fertilizes the polar nuclei, forming a ________ endosperm. The _____ will eventually develop into the sporophyte. How did Sulla help turn the Roman Republic into the Roman Empire? He angered the Senate by proposing that Rome divide public lands among the returning military and their families. He replaced the elected consuls and the Senate with an empire that could be inherited by members of the ruler's family. He angered the Senate by convincing men to serve in wars by promising them land and citizenship. He restructured the city's politics and strengthened the Senate's power. 4Ms. Thorson bought a piece of lace that was 3 feet long. She cuts off as many pieces as possible that are 9 inches long. Will Ms. Thorson have any lace left over? Show and explain your work. Select the correct verbal form to complete this sentence.Nosotros no ______ la verdad en el juicio.dijimosdecimosdejimosdicimos Compare The Epic of Gilgamesh and Homer's Iliad, focusing on characters, narrative, and themesWhat were the key differences in the Mesopotamian and Archaic Greek outlooks as exemplified by these texts? Which nursing action is applicable to the psychomotor domain of learning when conducting a teaching session for breastfeeding mothers Which U.S. territory became the first state west of the Rocky Mountains due to its rapid increase in population during the mid-1800s? Utah Oregon Nevada California Find g o for the following reaction, using h o f and s o values. H2(g) i2(s) 2 hi(g) Find the distance between (1, 2) and (-3, 1). Show your work and round your answer to the nearest tenth, ifnecessary. What is the area of a sector with a central angle of 107 radians and a radius of 18. 4 m? Use 3. 14 for and round your final answer to the nearest hundredth. Enter your answer as a decimal in the box. M. Suggest some ways in which you might determine whether a particular water solution contains an acid or a base. Which of the following is a typical product of nuclear fission? a. oxygen b. carbon dioxide c. radiation d. water please select the best answer from the choices provided a b c d Could the following side lengths make a right triangle?28, 53, 45 Porque es importante la hidratacion o hidratarnos?